Difference between revisions of "2015 AMC 10A Problems/Problem 21"

(Solution)
(Solution 2)
Line 15: Line 15:
  
 
==Solution 2==
 
==Solution 2==
Drop altitudes down from <math>C</math> and <math>D</math>, which will both hit a point we'll call <math>T</math>. Because both triangle <math>ABC</math> and triangle <math>ABD</math> are 3-4-5 triangles, <math>CT = DT = \dfrac{3\cdot4}{5} = 12/5</math>. Because <math>CT^{2} + DT^{2} = (\frac{12}{5})^{2}\cdot2 = (\frac{12}{5}\cdot\sqrt{2})^{2} = CD^{2}</math>, it follows that the <math>\angle CTD = \dfrac{\pi}{2}</math>, which means that planes <math>ABC</math> and <math>ABD</math> are perpendicular to each other. It follows that we can treat <math>ABC</math> as the base, and <math>CT</math> as the height. Thus the desired volume is <cmath>V = \dfrac{1}{3} Bh = \dfrac{1}{3}\cdot[ABC]\cdot CT = \dfrac{1}{3} \cdot 6 \cdot \dfrac{12}{5} = \dfrac{24}{5}</cmath> which is answer <math>\boxed{\textrm{C}}</math>.
+
Drop altitudes of triangle <math>ABC</math> and triangle <math>ABD</math> down from <math>C</math> and <math>D</math>, respectively. Both will hit the same point, which we'll call <math>T</math>. Because both triangle <math>ABC</math> and triangle <math>ABD</math> are 3-4-5 triangles, <math>CT = DT = \dfrac{3\cdot4}{5} = 12/5</math>. Because <math>CT^{2} + DT^{2} = (\frac{12}{5})^{2}\cdot2 = (\frac{12}{5}\cdot\sqrt{2})^{2} = CD^{2}</math>, it follows that the <math>\angle CTD = \dfrac{\pi}{2}</math>, which means that planes <math>ABC</math> and <math>ABD</math> are perpendicular to each other. It follows that we can treat <math>ABC</math> as the base, and <math>CT</math> as the height. Thus the desired volume is <cmath>V = \dfrac{1}{3} Bh = \dfrac{1}{3}\cdot[ABC]\cdot CT = \dfrac{1}{3} \cdot 6 \cdot \dfrac{12}{5} = \dfrac{24}{5}</cmath> which is answer <math>\boxed{\textrm{C}}</math>.
  
 
== See Also ==
 
== See Also ==

Revision as of 00:20, 5 February 2015

The following problem is from both the 2015 AMC 12A #16 and 2015 AMC 10A #21, so both problems redirect to this page.

Problem

Tetrahedron $ABCD$ has $AB=5$, $AC=3$, $BC=4$, $BD=4$, $AD=3$, and $CD=\tfrac{12}5\sqrt2$. What is the volume of the tetrahedron?

$\textbf{(A) }3\sqrt2\qquad\textbf{(B) }2\sqrt5\qquad\textbf{(C) }\dfrac{24}5\qquad\textbf{(D) }3\sqrt3\qquad\textbf{(E) }\dfrac{24}5\sqrt2$

Solution 1

Let the midpoint of $CD$ be $E$. We have $CE = \dfrac{6}{5} \sqrt{2}$, and so by the Pythagorean Theorem $AE = \dfrac{\sqrt{153}}{5}$ and $BE = \dfrac{\sqrt{328}}{5}$. Because the altitude from $A$ of tetrahedron $ABCD$ passes touches plane $BCD$ on $BE$, it is also an altitude of triangle $ABE$. The area $A$ of triangle $ABE$ is, by Heron's Formula, given by

\[16A^2 = 2a^2 b^2 + 2b^2 c^2 + 2c^2 a^2 - a^4 - b^4 - c^4 = -(a^2 + b^2 - c^2)^2 + 4a^2 b^2.\] Substituting $a = AE, b = BE, c = 5$ and performing huge (but manageable) computations yield $A^2 = 18$, so $A = 3\sqrt{2}$. Thus, if $h$ is the length of the altitude from $A$ of the tetrahedron, $BE \cdot h = 2A = 6\sqrt{2}$. Our answer is thus \[V = \dfrac{1}{3} Bh = \dfrac{1}{3} h \cdot BE \cdot \dfrac{6\sqrt{2}}{5} = \dfrac{24}{5},\] and so our answer is $\textbf{(C)}$.


Solution 2

Drop altitudes of triangle $ABC$ and triangle $ABD$ down from $C$ and $D$, respectively. Both will hit the same point, which we'll call $T$. Because both triangle $ABC$ and triangle $ABD$ are 3-4-5 triangles, $CT = DT = \dfrac{3\cdot4}{5} = 12/5$. Because $CT^{2} + DT^{2} = (\frac{12}{5})^{2}\cdot2 = (\frac{12}{5}\cdot\sqrt{2})^{2} = CD^{2}$, it follows that the $\angle CTD = \dfrac{\pi}{2}$, which means that planes $ABC$ and $ABD$ are perpendicular to each other. It follows that we can treat $ABC$ as the base, and $CT$ as the height. Thus the desired volume is \[V = \dfrac{1}{3} Bh = \dfrac{1}{3}\cdot[ABC]\cdot CT = \dfrac{1}{3} \cdot 6 \cdot \dfrac{12}{5} = \dfrac{24}{5}\] which is answer $\boxed{\textrm{C}}$.

See Also

2015 AMC 10A (ProblemsAnswer KeyResources)
Preceded by
Problem 20
Followed by
Problem 22
1 2 3 4 5 6 7 8 9 10 11 12 13 14 15 16 17 18 19 20 21 22 23 24 25
All AMC 10 Problems and Solutions
2015 AMC 12A (ProblemsAnswer KeyResources)
Preceded by
Problem 15
Followed by
Problem 17
1 2 3 4 5 6 7 8 9 10 11 12 13 14 15 16 17 18 19 20 21 22 23 24 25
All AMC 12 Problems and Solutions

The problems on this page are copyrighted by the Mathematical Association of America's American Mathematics Competitions. AMC logo.png